a professor at the university of florida wanted to determine if offering video tutorials for the course software would increase student engagement. the engagement ratings are below for a random sample of 5 students before and after implementing the course change. ratings were on a scale between 0 and 50. the higher scores translated to higher student engagement score. student before after 1 30 40 2 20 40 3 32 37 4 43 46 5 48 44 what is the test statistic for the wilcoxon signed rank test? group of answer choices 1

Answers

Answer 1

According to the information, he test statistic for this sample is 5.

How to calculate the test statistic for the Wilcoxon signed-rank test?

To calculate the test statistic for the Wilcoxon signed-rank test, we need to calculate the differences between the "before" and "after" engagement ratings and rank them in order of their absolute values.

Student Before After Difference Absolute Difference Rank

1 30 40 10 10 1

2 20 40 20 20 2

3 32 37 5 5 3

4 43 46 3 3 4

5 48 44 -4 4 5

The sum of the ranks for the positive differences is 1 + 2 + 3 + 4 = 10, and the sum of the ranks for the negative differences is 5.

The smaller of the two sums (in this case, the sum of the ranks for the negative differences) is the test statistic for the Wilcoxon signed-rank test.

Therefore, the test statistic for this sample is 5.

Learn more about statistic in: https://brainly.com/question/29093686

#SPJ1


Related Questions

Please help!! URGENT!! I don’t understand

Answers

Answer: 14%

Step-by-step explanation:

Our formula is i=prt but we're trying to find the rate (r) so we'll rearrange it to become r=i/pt.

i = $245

p = $7000

t = 3/12= 0.25 years

[tex]r=\frac{245}{(7000)(0.25)} \\r = 0.14[/tex]

Julian goes to a store an buys an item that costs � x dollars. He has a coupon for 20% off, and then a 4% tax is added to the discounted price. Write an expression in terms of � x that represents the total amount that Julian paid at the register.

Answers

The expression that represents the total amount that Julian paid at the register in terms of x is 0.84x.

What is Percentage?

Percentage is a way of expressing a proportion or fraction as a quantity out of 100. The word "percent" means "per hundred," so percentages are often denoted by the symbol %, which represents one part in a hundred.

The first step is to find the discounted price after the 20% discount. This can be found by multiplying the original price by (1 - 0.2), which represents a 20% reduction.

Discounted price = x - 0.2x = 0.8x

Next, a 4% tax is added to the discounted price. This can be found by multiplying the discounted price by (1 + 0.04), which represents a 4% increase.

Total amount paid = (0.8x) * (1 + 0.04) = 0.84x

Therefore, the expression that represents the total amount that Julian paid at the register in terms of x is 0.84x.

To learn more about Percentage visit the link:

https://brainly.com/question/24877689

#SPJ1

Use the table to answer the question that follows. ROR Portfolio 1 Portfolio 2 Portfolio 3
8. 9% $850 $1,050 $1,175
−5. 1% $2,425 $1,950 $550
12. 4% $280 $1,295 $860
4. 2% $1,400 $745 $550
0. 9% $2,330 $1,050 $2,000


Using technology, calculate the weighted mean of the RORs for each portfolio. Based on the results, which list shows a comparison of the overall performance of the portfolios, from best to worst?

Answers

The weighted mean of the RORs for each portfolio, from best to worst, is:

Portfolio 3: 4.36%

Portfolio 2: 3.2%

Portfolio 1: 0.94%

To calculate the weighted mean of the RORs for each portfolio, we need to multiply each rate of return by its corresponding portfolio value, sum the results, and divide by the total value of the portfolios:

Weighted Mean ROR = sum of value / total amount

sum of value = portfolio value x rate of return

For Portfolio 1:

(0.089 x 850) + (-0.051 x 2425) + (0.124 x 280) + (0.042 x 1400) + (0.009 x 2330)

Sum of value = 75.65 - 123.68 + 34.72 + 58.8 + 20.97

                      = 61.46.

Weighted Mean ROR = 61.46 / (850 + 2425 + 280 + 1400 + 2330)

                                    = 61.46 / 7285

                                    = 0.0094 or 0.94% (third)

For Portfolio 2:

(0.089 x 1050) + (-0.051 x 1950) + (0.124 x 1295) + (0.042 x 745) + (0.009 x 1050)

Sum of value = 93.45 - 99.45 + 160.58 + 31.29 + 9.45

                      = 195.32.

Weighted Mean ROR = 195.32 / (1050 + 1950 + 1295 + 745 + 1050)

                                    = 195.32 / 6090

                                    = 0.032 or 3.2% (second)

For Portfolio 3:

(0.089 x 1175) + (-0.051 x 550) + (0.124 x 860) + (0.042 x 550) + (0.009 x 2000)

Sum of value = 104.58 - 28.05 + 106.64 + 23.1 + 18

                      = 224.27.

Weighted Mean ROR = 224.27 / (1175 + 550 + 860 + 550 + 2000)

                                    = 224.27 / 5,135

                                    = 0.0436 or 4.36% (first)

To compare the overall performance of the portfolios, we can rank them based on their weighted mean RORs, from highest to lowest:

Portfolio 3: 4.36%

Portfolio 2: 3.2%

Portfolio 1: 0.94%

Therefore, the overall performance of the portfolios, from best to worst, is Portfolio 3, Portfolio 2, and Portfolio 1.

To know more about portfolio

https://brainly.com/question/29333981

#SPJ4

HELP ME PLS
1. If the diameter of a circle is segment AB where Point A is located at (-3, 6), and Point B located at (-8,-1), what is the diameter of the circle?

2√2
√146
√74
74

Answers

The diameter of the circle is √74. Answer: C.

What is circle?

A circle is a geometric shape that consists of all points in a plane that are equidistant from a fixed point called the center. It can also be defined as the set of points that are a fixed distance (called the radius) away from the center point.

To find the diameter of the circle, we need to find the distance between points A and B, which will give us the length of the diameter.

Using the distance formula, we have:

d = √[(x2 - x1)² + (y2 - y1)²]

where (x1, y1) = (-3, 6) and (x2, y2) = (-8, -1).

Plugging in the values, we get:

d = √[(-8 - (-3))² + (-1 - 6)²]

= √[(-5)² + (-7)²]

= √[25 + 49]

= √74

Therefore, the diameter of the circle is √74. Answer: C.

To learn more about circle from the given link:

https://brainly.com/question/29142813

#SPJ1

thompson and thompson is a steel bolts manufacturing company. their current steel bolts have a mean diameter of 137 millimeters, and a variance of 49 . if a random sample of 48 steel bolts is selected, what is the probability that the sample mean would differ from the population mean by greater than 3 millimeters? round your answer to four decimal places.

Answers

The probability that the sample mean would differ from the population mean by greater than 3 millimetres is 0.0033 + 0.0033 = 0.0066, rounded to four decimal places.

We are given that the population mean diameter of the steel bolts manufactured by Thompson and Thompson is μ = 137 millimeters and the variance is = 49.

We need to find the probability that the sample mean would differ from the population mean by greater than 3 millimeters.

The standard deviation of the sample means is given by the formula:

[tex]\sigma_{\bar{x}} = \frac{\sigma}{{\sqrt{n}}}[/tex]

Substituting the given values, we have:

[tex]\sigma \bar{x}=\frac{\sqrt{49}}{\sqrt{48}}=1.118[/tex]

To find the probability that the sample mean would differ from the population mean by greater than 3 millimeters,

we need to calculate the z-score:

[tex]z=\frac{(\bar{x}-\mu)}{ \sigma _{\bar{x}}}[/tex]

Substituting the given values, we have:

[tex]z=\frac{\bar{x}-137}{1.118}[/tex]

We want to find the probability that |z| > 3/1.118 = 2.683.

Using a standard normal distribution table, we find that the probability of z > 2.683 is 0.0033.

Since this is a two-tailed test, the probability of z < -2.683 is also 0.0033.

For more questions on standard normal distribution table

https://brainly.com/question/1846009

#SPJ11

The function V(t) = 30000(0.85) value V(t) represents the values v(t) of Nancy's car after t years. What is the depreciation rate of Nancy's car?​

Answers

The depreciation rate of Nancy is 15%.

What is known by depreciation?

Depreciation is the reduction in the value of an asset over time due to wear and tear, obsolescence, or other factors. It is a non-cash expense that is recorded on a company's income statement, which reflects the decrease in the asset's value during its useful life. Depreciation is commonly used in accounting to allocate the cost of an asset over its useful life, and it helps businesses to accurately reflect the true value of their assets on their financial statements.

Define function?

In mathematics, a function is a relation between a set of inputs and a set of possible outputs with the property that each input is related to exactly one output.

The given function V(t) = [tex](30000*0.85)^{t}[/tex]represents the value of Nancy's car after t years.

Depreciation rate is the rate at which the value of an asset decreases over time. In this case, the depreciation rate of Nancy's car is 1 - 0.85 = 0.15 or 15%.

Learn more about depreciation here:

https://brainly.com/question/17827672

#SPJ1

true or false: at the initial examination in the framingham study, coronary heart disease was found in 5 per 1000 men ages 30-44, and in 5 per 1000 women ages 30-44. the inference that in this age group men and women have an equal risk of getting coronary heart disease is incorrect because the data are prevalence data and not incidence data. group of answer choices true false

Answers

The inference is that,  Incorrect because of a failure to distinguish between incidence and prevalence.

What is inference drawn from statistics?

Utilising data analysis to determine characteristics of a probability distribution at play, statistical inference is the process. Through the use of estimation and hypothesis testing, inferential statistical analysis infers characteristics of a population. It is presumed that a bigger population than the observed data set was sampled for this data collection.

What distinguishes random assignment from random sampling inferences?

As a method of choosing participants for your study's sample, random selection or random sampling can be used. The sample can be divided into control and experimental groups using random assignment, however.

Learn more about inference

brainly.com/question/29774121

#SPJ1

The complete question is -

At the initial examination in the Framingham study, coronary heart disease was found in 5 per 1,000 men aged 30yrs to 44yrs and in 5 per 1,000 women aged 30yrs to 44yrs. The inference that in this age group men and women have an equal risk of developing coronary heart disease is Selected Answer: A. Correct Answers: A. Correct B. Incorrect because of a failure to distinguish between incidence and prevalence C. Incorrect because a proportionate ratio is used when a rate is required to support the inference D. Incorrect because of failure to recognize a possible cohort phenomenon E. Incorrect because there is no control or comparison group.

There are 90 pens in a box and 36 of them are red and the rest of them are black What is the ratio of the red pens to black pens

Answers

If there are 90 pens in a box and 36 of them are red and the rest of them are black, the ratio of red pens to black pens in the box is 2:3.

The ratio of red pens to black pens in the box can be expressed as a fraction or as a simplified ratio. To do this, we need to determine the number of black pens in the box.

We know that there are 90 pens in total, and 36 of them are red. Therefore, the number of black pens can be found by subtracting the number of red pens from the total:

90 pens - 36 red pens = 54 black pens

Now we know that there are 54 black pens and 36 red pens in the box. To express this as a ratio, we can simplify it by dividing both numbers by their greatest common factor (GCF), which in this case is 18:

36/18 : 54/18

Simplifying further, we get:

2:3

This means that for every 2 red pens, there are 3 black pens in the box.

To learn more about ratio click on,

https://brainly.com/question/7414039

#SPJ4

the major flaw of the linear probability model is that a. the actuals can only be 0 and 1, but the predicted are almost always different from that. b. the regression r2 cannot be used as a measure of fit. c. people do not always make clear-cut decisions. d. the predicted values can lie above 1 and below 0.

Answers

The major flaw of the linear probability model is, Option d, which allows predicted values to range from above 1 to below 0

The binary dependent variable, which accepts values of 0 and 1, and the independent variables are assumed to have a linear relationship under the linear probability model.

The anticipated values from the linear probability model, however, can range from 0 to 1, and in some circumstances, they can be either above or below 0. This goes against the dependent variable's probabilistic character and can produce inaccurate forecasts.

To know more about Linear probability, visit,

https://brainly.com/question/28013612

#SPJ4

A small tree that is 8 feet tall casts a 4-foot shadow, while a building that is 24 feet tall casts a shadow in the same direction. Determine the length of the building's shadow.

6 feet
12 feet
18 feet
48 feet

Answers

Answer:

Using similar triangles, we can set up a proportion:

(tree height)/(tree shadow) = (building height)/(building shadow)

Plugging in the given values, we get:

8/4 = 24/x

Solving for x, we get:

x = (24 x 4)/8 = 12 feet

Therefore, the length of the building's shadow is B. 12 feet.

From a piece of cardboard that is 32 inches by 12 inches, you are making a box by cutting equal squares at each corner and folding it up. What size should the squares be for the volume of the box to be maximal? Find x

Answers

For the maximum volume of , the value of x must be smaller that is x = 2.90 inch. Size of square = 8.41 sq. in.

Explain about maxima and minima:

The issue is an application of maxima and minima, where the unknown quantity is located using the first derivative. Keep in mind that the height of a box is equal to a side of the cut square when the sides are bent upward. Moreover, the box formula's volume is

Volume = length * width * height

Given:

Length of rectangle cardboard l = 32 inchesWidth of rectangle cardboard w = 12 inchesLength of the side of each square - x inxhes.

Volume of new box = (32 - 2x)(12 - 2x)x

Rearranging terms:

V = 4x³ - 88x² + 384x

For maximum volume, V' = 0 (first derivative)

V' = 12x² - 167x + 384

V' = 0

12x² - 167x + 384 = 0

Solve using quadratic formula:

x = [-b ± √(b² - 4ac)] / 2a

a = 12 , b = - 167 , c = 384

x = [167 ± √(167² - 4*384*12)] / 2*12

x =  [167 ± 7√193] / 24

Now,

x =  [167 + 7√193] / 24

x = 11.01 inch

and,

x = [167 - 7√193] / 24

x = 2.90 inch

For the maximum volume of , the value of x must be smaller that is x = 2.90 inch.

Size of square = x²

Size of square = 2.90²

Size of square = 8.41 sq. in.

Know more about the maxima and minima

https://brainly.com/question/29502088

#SPJ1

Please help offering 15 points!!

Answers

Answer: C 70%

Step-by-step explanation:

First you need to add up how many students are participating in the study. 4+6+12+8+4=34. There are 24 people who own more than 5 video games. This means that it is 24/34. This equals about 70%

From the top of a 120-foot-high tower, an air traffic controller observes an airplane on the runway at an angle of depression of 19°. How far from the base of the tower is the airplane? Round to the nearest tenth.

1. 126.9 ft
2. 368.6 ft
3. 41.3 ft
4. 348.5 ft​

Answers

Using a trigonometric relation we can see that the correct option is the last one;

4. 348.5 ft​

How to find the distance?

We have a right triangle, so we can use trigonometric relations.

The angle at the top of the triangle is a, and it must solve the equation:

a + 19° = 90°

a = 90° - 19° = 71°

Now we can use the trigonometric relation:

tan(a) = (opposite cathetus)/(adjacent cathetus)

REplacing what we know:

tan(71) = ?/120 ft

120ft*tan(71) = ? = 348.5ft

That is the distance.

Learn more about right triangles at:

https://brainly.com/question/2217700

#SPJ1

Solve the Simple Interest
Ruby contributes 12% of the total cost of her individual health care. This is a $57.50 deduction from each of her biweekly paychecks. What is the total value of her individual coverage for the year? Find its employer share.

Answers

Using simple interest we know the Total value of annual health coverage is $9000.

What is simple interest?

Borrowers must pay lenders simple interest as a fee in exchange for a loan.

Compound interest is excluded from the calculation and just the original principal is used.

Simple interest applies to all loans, not just specific ones.

Additionally, it refers to the kind of interest that banks give their customers on their savings accounts.

So, complete yearly health coverage calculation

Let x be the total annual healthcare budget.

Ruth contributes 18% of the entire cost of healthcare, or $67.50 for two weeks. (ie 15 days)

So, total paid for one month = 67.5 x 2 = 135

The total amount paid for the entire year is 135 x 12 = 1620.

She foots 18% of the overall annual health care costs, as was already established.

Then,

18% x = 1620

18 x / 100 = 1620

x = (1620 x 100) / 18

= $9000

Therefore, using simple interest we know the Total value of annual health coverage is $9000.

Know more about simple interest here:

https://brainly.com/question/25793394

#SPJ1

Correct question:

Ruth contributes 18% of the total cost of her individual health care. This is a $67.50 deduction from each of her biweekly paychecks. What is the total value of her individual coverage for the year?

Urgent, please help!

Answers

It has been proved that the lines MN and PR are parallel because they have the same slope.

How to identify the slope of parallel lines?

We are given the coordinates of the points P, Q and R as:

P(-3, -6)

Q(1, 4)

R(5, -2)

Since M is the midpoint of PQ, we have:

M = (-3 + 1)/2, (-6 + 4)/2

M = (-1, -1)

N is the midpoint of QR and so:

N = (1 + 5)/2, (4 - 2)/2

N = (3, 1)

Slope of MN = (1 + 1)/(3 + 1) = 1/2

Slope of PR = (-2 + 6)/(5 + 3)

Slope of QR = 4/8 = 1/2

Both slopes are equal and as such they are parallel lines

Read more about parallel lines at: https://brainly.com/question/29056723

#SPJ1

two fair 6-sided dice are rolled. what is the probability that the sum of the two numbers on the dice is greater than 10?

Answers

The probability of rolling a sum greater than 10 is 4/36, which simplifies to 1/9, or approximately 0.1111.

There are 36 possible outcomes when rolling two 6-sided dice. Each outcome is equally likely.

To find the probability of rolling a sum greater than 10, we need to count the number of outcomes where the sum is greater than 10, and then divide by the total number of outcomes.

The outcomes where the sum is greater than 10 are:

4 + 6 = 10

5 + 6 = 11

6 + 5 = 11

6 + 6 = 12

So there are 4 possible outcomes where the sum is greater than 10.

For similar question on probability.

https://brainly.com/question/28021875

#SPJ11

find the volume of a cylinder with a diameter of 22 yd and a height of 7 yd

Answers

V=π(d

2)2h=π·(20.12

2)2·6.4≈2034.42618m

A coordinate plane. The x- and y-axes each scale by one. A graph of a line intersects the points negative four, negative three and negative one, negative two. A coordinate plane. The x- and y-axes each scale by one. A graph of a line intersects the points negative four, negative three and negative one, negative two. What is the slope of the line?

Answers

The slope of the line on this graph is equal to 1/3.

How to calculate or determine the slope of a line?

In Mathematics and Geometry, the slope of any straight line can be determined by using the following mathematical equation;

Slope (m) = (Change in y-axis, Δy)/(Change in x-axis, Δx)

Slope (m) = rise/run

Slope (m) = (y₂ - y₁)/(x₂ - x₁)

By substituting the given data points into the slope formula, we have the following;

Slope (m) = (-2 - (-3))/(-1 - (-4))

Slope (m) = (-2 + 3)/(-1 + 4)

Slope (m) = 1/3

Based on the graph, the slope is the change in y-axis with respect to the x-axis and it is equal to 1/3.

Read more on slope here: brainly.com/question/3493733

#SPJ1

The slope of the coordinate  line is 1/3.

Coordinate plane calculation.

To find the slope of the line that passes through the points (-4, -3) and (-1, -2), we use the slope formula:

slope = (change in y) / (change in x)

The change in y is -2 - (-3) = 1, and the change in x is -1 - (-4) = 3. Therefore, the slope is:

slope = 1 / 3

So the slope of the line is 1/3.

Learn more about coordinate plane below.

https://brainly.com/question/17206319

#SPJ1

Cheyenne has a part-time job at which she gets paid by the hour. The table below shows how much money Cheyenne can earn for working various amounts of hours.

How many hours, h, must Cheyenne work to earn $56.00?

Answers

Answer:

Can i see the full answer so I can help you?

Step-by-step explanation:

7 hours
Cheyenne is making $8 an hour based on the chart.
8x4 =32
8x5= 40
8x7=56
8x10=80
8x12=96

Mr. Yara has 200 acres of land available to plant for three types of fruit trees (lemon, peach and

cherry). Based on the market survey, lemon has a selling price of $2 per kg; peach has a selling

price of $1. 5 per kg and cherry has a selling price of $4 per kg. The average yield per acre for

lemon, peach and cherry is 4, 6 and 2 tonne, respectively. The fertilizer requires per acre for

lemon, peach and cherry is 100, 100 and 50 kg, respectively. An acre of lemon and cherry

requires 10 labour hours each whereas an acre of peach requires 12 labour hours. Maximum

availability of labour is 20,000 hours. The cost of fertilizer is $2 per kg and the cost of labour is

$40 per hour. What is the optimal allocation of acre to the three types of fruit trees?

[1 tonne = 1000kg]


(a) Formulate a linear programming model to maximize the profit.


(b) Find the optimal solution by using simplex algorithm.

Answers

(a) The linear programming model to maximize the profit is written as,

Maximize Z = 2(4x1)(100) + 1.5(6x2)(100) + 4(2x3)(1000) - 2(100x1 + 100x2 + 50x3) - 40(10x1 + 12x2)

(b) The optimal solution is x1 = 0, x2 = 8/3, x3 = 8, x4 = 0, and the maximum profit is $40.

(a) Linear programming model:

Let x1, x2, and x3 be the number of acres allocated to lemon, peach, and cherry trees, respectively. Then, the objective function we want to maximize is:

Maximize Z = 2(4x1)(100) + 1.5(6x2)(100) + 4(2x3)(1000) - 2(100x1 + 100x2 + 50x3) - 40(10x1 + 12x2)

The first term represents the revenue from selling lemons, the second term represents the revenue from selling peaches, the third term represents the revenue from selling cherries, the fourth term represents the cost of fertilizer, and the last term represents the cost of labor.

The constraints are:

x1 + x2 + x3 ≤ 200 (total available land)

10x1 + 12x2 + 10x3 ≤ 20000 (total available labor hours)

x1, x2, x3 ≥ 0 (non-negativity constraints)

(b) Simplex algorithm:

Convert the linear programming model into standard form by introducing slack variables and expressing all variables in terms of these variables.

Maximize Z = 2(4x1)(100) + 1.5(6x2)(100) + 4(2x3)(1000) - 2(100x1 + 100x2 + 50x3) - 40(10x1 + 12x2)

Subject to:

x1 + x2 + x3 + x4 = 200

10x1 + 12x2 + 10x3 + x5 = 20000

x1, x2, x3, x4, x5 ≥ 0

In this case, the entering variable is x1, as it has the highest coefficient in the objective function.

Step 5 (continued):

1.6 0.1 0 1 -48 32 <- Z

0.4 0.9 1 1/10 0 40 <- x1

8.4 3.3 0 -1/10 1 1840 <- x5

The entering variable is x3, as it has the highest coefficient in the objective function.

The leaving variable is x1, as it has the smallest ratio of RHS to its coefficient in the x3 column.

x2 x1/12 x3 x5/120 x4 RHS

2.5 1/5 0 -1/60 0 40 <- Z

0.5 -1/5 1 1/60 0 8 <- x3

0.5 2/15 0 1/60 1/10 8/3 <- x2

To know more about linear programming model here

https://brainly.com/question/28036767

#SPJ4

Thabo opens an investment account and invest an amount of money at 8. 00% interest per year, compounded monthly. After a number of years, he has accumulated an amount of R 7 365. 00 in the account. The investment earned R2 000. 00 interest in this period. If the accumulated amount is left in the account with the same interest rate, for another period that is one year longer than the first period, the accumulated amount in the account will then be?

Answers

The accumulated amount in the account at the end of the second period will be 8,829.71.

We can use the formula for compound interest to solve this problem. The formula is,

A = P(1 + r/n)^(nt)

where A is the accumulated amount, P is the principal (initial amount invested), r is the interest rate (as a decimal), n is the number of times the interest is compounded per year, and t is the time (in years) for which the money is invested.

Let's use this formula to find the initial principal, P

7,365 = P(1 + 0.08/12)^(12*number of years)

We can solve for P by dividing both sides by the right-hand side and simplifying,

P = 7,365 / (1 + 0.08/12)^(12*number of years)

Now we know that the initial principal was P, and it earned R2 000.00 in interest during the first period. Therefore, the accumulated amount at the end of the first period was,

A1 = P + R2 000.00

A1 = P + P(0.08/12)

A1 = P(1 + 0.08/12)

Now, we want to find the accumulated amount at the end of the second period, which is one year longer than the first period. We can use the same formula as before, but with a time of (number of years + 1),

A2 = P(1 + 0.08/12)^(12*(number of years + 1))

We know that A1 = P(1 + 0.08/12), so we can substitute this into the formula for A2,

A2 = A1(1 + 0.08/12)^(12)

A2 = (P(1 + 0.08/12))(1 + 0.08/12)^(12)

A2 = P(1 + 0.08/12)^13

Now we can substitute the expression we found for P earlier,

A2 = (7,365 / (1 + 0.08/12)^(12*number of years))(1 + 0.08/12)^13

A2 = 7,365(1 + 0.08/12)^(12*number of years + 13)

A2 = 7,365(1.007)^((12*number of years) + 13)

A2 = 8,829.71

To learn more about accumulated amount here:

https://brainly.com/question/20388730

#SPJ4

A football team consists of:
• 10 sixth graders
• 14 seventh graders
• 16 eighth graders

A student on the team will be randomly chosen to participate in the coin toss each of the 40
games of the season.
What is a reasonable prediction for the number of times a sixth or seventh grader will be
chosen?

Answers

A reasonable prediction for the number of times a sixth or seventh grader will be chosen is 24 out of the 40 games.

What is reasonable prediction?

A reasonable prediction is a prediction made with a reasonable degree of accuracy or likelihood, based on available information and knowledge. It is based on facts, past events, and logical assumptions, and is not based on conjecture or guesswork. Reasonable predictions can be made about future events, trends, and outcomes, and can be used to inform decisions, plans, and strategies.

The total number of sixth and seventh graders on the team is:

10 sixth graders + 14 seventh graders = 24 students

The total number of students on the team is:

10 sixth graders + 14 seventh graders + 16 eighth graders = 40 students

To find the expected number of times a sixth or seventh grader will be chosen in the coin toss, we can use the proportion of sixth and seventh graders to the total number of students:

(expected number of times) = (proportion of sixth and seventh graders) * (total number of coin tosses)

(expected number of times) = (24/40) * 40

(expected number of times) = 24

Therefore, a reasonable prediction for the number of times a sixth or seventh grader will be chosen is  24.

To learn more about reasonable prediction

brainly.com/question/31255693

#SPJ1

explain why we call the national halothane study an observational study rather than an experiment, even though it compared the results of using different anesthetics in actual surgery.in order to be an experiment, the subjects would have to be randomly selected from the population. however, these are hospital patients who all have some disease or condition and have not been randomly selected from the population, which includes both healthy and sick people.in order to be an experiment, the treatments (choice of anesthetic) would have to be randomly assigned. instead, a patient's anesthetic is selected by his or her doctor(s).there is not enough information to say for sure, but it is safer to assume that it is only an observational study, so that we are not overconfident about the results.actually, it is an experiment and not an observational study.

Answers

We the best reason behind why we call National halothane study is an observational study rather than an experiment is: " In order to be an experiment, the treatments (choice of anesthetic) would have to be randomly assigned." So, option( b) is right one.

We have data from the National Halothane Study that correlates with important research into the safety of anesthetics used in surgery. The above shows the death rate and aesthetic. There is a relationship between the anesthetic used and the death of the patient. The data was collected after that the anesthesia was administered is started. In the observational study, interesting observer in the subjects & any treatment that the subjects recive are beyond the council of the investigators. Here the motion of anaesthesia is anesthetics used her determined by the doctors and the investigators are simply observing. So, it is an observational study due to because in order to be an experiment, the treatments (choice of anesthetic) would have to be randomly assigned. Instead, a patient's anesthetic is selected by his or her doctor(s).

For more information about observational study, visit :

https://brainly.com/question/14393640

#SPJ4

Complete question:

The National Halothane Study was a major investigation of the safety of anesthetics used in surgery. Records of over 850,000 operations performed in 34 major hospitals showed the following death rates for four common anesthetics:27 Anesthetic A Death rate B CD 1.7% 1.7% 3.4% 1.9% There is a clear association between the anesthetic used and the death rate of patients. Anesthetic C appears dangerous.

a) in order to be an experiment, the subjects would have to be randomly selected from the population. however, these are hospital patients who all have some disease or condition and have not been randomly selected from the population, which includes both healthy and sick people.

b) in order to be an experiment, the treatments (choice of anesthetic) would have to be randomly assigned. instead, a patient's anesthetic is selected by his or her doctor(s).

c) there is not enough information to say for sure, but it is safer to assume that it is only an observational study, so that we are not overconfident about the results.

d) actually, it is an experiment and not an observational study.

a rectangular poster is to contain 392 square inches of print. the margins at the top and bottom of the poster are to be 2 inches, and the margins on the left and right are to be 1 inch. what should the dimensions of the poster be so that the least amount of poster is used?

Answers

The dimensions of the poster be so that the least amount of poster is used are A = 6L + 4W + 412.

Let the length and width of the printable area of the poster be L and W, respectively. Then, the total dimensions of the poster can be expressed as L + 2(2) and W + 2(1), since there are 2-inch margins at the top and bottom, and 1-inch margins on the left and right.

We know that the area of the printable area of the poster is 392 square inches. Therefore, we can write the equation: LW = 392

We want to minimize the total area of the poster, which is given by:

A = (L + 2(2))(W + 2(1)) = (L + 4)(W + 2)

Expanding this expression, we get:

A = LW + 2L + 4W + 8

Substituting the equation for LW, we get:

A = 392 + 2L + 4W + 8

Simplifying, we get:

A = 2L + 4W + 400

To minimize this expression, we can take the partial derivatives with respect to L and W and set them equal to zero:

[tex]∂A/∂L = 2 = 0 => L = 0[/tex]

[tex]

∂A/∂W = 4 = 0 => W = -100[/tex]

These values do not make sense in the context of the problem. Therefore, we can conclude that the dimensions of the poster that minimize the amount of poster used cannot be found using this method.

Instead, we can use the fact that the printable area of the poster has a fixed area of 392 square inches, and that the margins have fixed dimensions. We can express the area of the poster as:

A = (L + 4)(W + 2) = LW + 4L + 2W + 8

Substituting the equation for LW, we get:

A = 392 + 4L + 2W + 8

Simplifying, we get:

A = 4L + 2W + 400

To minimize this expression, we can again take the partial derivatives with respect to L and W and set them equal to zero:

[tex]∂A/∂L = 4 = 0 => L = 0[/tex]

[tex]∂A/∂W = 2 = 0 => W = -200[/tex]

These values do not make sense in the context of the problem. Therefore, we can conclude that the dimensions of the poster that minimize the amount of poster used cannot be found using this method either.

We can try a different approach. We can use the fact that the printable area of the poster has a fixed area of 392 square inches, and that the total area of the poster is given by:

A = (L + 4)(W + 2) + 2(L + 4) + 2(W + 2)

Expanding this expression, we get:

A = LW + 6L + 4W + 20

Substituting the equation for LW, we get:

A = 392 + 6L + 4W + 20

Simplifying, we get: A = 6L + 4W + 412

To minimize this expression, we can take the partial derivatives with respect to L and W and set them equal to zero:

[tex]∂A/∂L = 6 = 0 => L = -2/3[/tex]

[tex]∂A/∂W = 4 = 0 => W = -3/2[/tex]

Learn more about dimensions here:

https://brainly.com/question/29581656

#SPJ4

Use the functions f(x) and g(x) to answer the question. F(x)=x²-16; g(x)=x+4


Which answer is equal to (f/g)(x)


A x+4,x=-4

B x+4,x=4

C x-4,x=4

D x-4,x=-4


**All eqaul signs in answers are "do not equal signs"**

Answers

Using the functions f(x) and g(x) as F(x)=x²-16; g(x)=x+4, (f/g)(x) = (x^2 - 16)/(x + 4) when x = -4. So, the correct answer is option D.

To find (f/g)(x), we need to divide f(x) by g(x). The notation (f/g)(x) is another way of representing the function f(x)/g(x).

So, we first need to find f(x)/g(x) by dividing f(x) by g(x):

f(x)/g(x) = (x^2 - 16)/(x + 4)

Now, we can plug in the values of x in each option and see which one gives us the correct function.

Option A: (f/g)(-4) = ((-4)^2 - 16)/(-4 + 4) = undefined

Option B: (f/g)(4) = (4^2 - 16)/(4 + 4) = -1

Option C: (f/g)(4 - 4) = (0^2 - 16)/(4 - 4) = undefined

Option D: (f/g)(-4 - 4) = ((-4)^2 - 16)/(-4 - 4) = 0

Therefore, the correct answer is option D: (f/g)(x) = (x^2 - 16)/(x + 4) when x = -4.

Note that options A and C give undefined values because they have x + 4 in the denominator, which makes the function undefined at x = -4 and x = 4, respectively.

To learn more about functions click on,

https://brainly.com/question/20317632

#SPJ4

Can someone help me fast with this please!?!?!

If s(d) represents the number of songs downloaded in a year d, what is the interpretation of s(2020) = 5,220,000.
A. 5,220,000 songs were downloaded in the year 2020.
B. There is not enough information to interpret the information.
C. In the year 2020 $5,220,000 was earned from downloaded songs.
D. 2,020 songs were downloaded at a cost of $5,220,000.

Answers

the correct answer is A.

5,220,000 songs were downloaded in the year 2020.

If s(d) represents the number of songs downloaded in a year d, what is the interpretation of s(2020) = 5,220,000?

The interpretation of s(2020) = 5,220,000 is "5,220,000 songs were downloaded in the year 2020".

Therefore, the correct answer is A. 5,220,000 songs were downloaded in the year 2020.

Interpretation refers to the process of explaining or giving meaning to information, data, or phenomena. It involves analyzing and making sense of information to draw conclusions or make decisions based on the findings.

Interpretation can involve subjective judgment and may depend on the perspective, assumptions, or biases of the interpreter.

to know more about interpretation

brainly.com/question/30932003

#SPJ1

please help with these assap will give brainlest!!

Answers

Answer:

below

Step-by-step explanation:

58.

(72 + 75 + 68 + 70 + 73 + 72 + 76 + 72 + 69 + 72 ) divided by 10 = 71.9

59.

order- 68, 69, 70, 72 , 72, 72, 72, 73, 75, 76

median = (72 + 72) divided by 2 = 72

61.

I dont know what MAD means

this is all I could do rn

what other patterns can you discover in the subway ridership dataset using calculations with multiple operators?

Answers

Subway ridership data can reveal useful patterns through calculations with multiple operators, informing decisions about infrastructure and scheduling to better serve riders' needs.

Why Subway ridership data reveal useful patterns through calculations?

The subway ridership dataset can reveal a wealth of information through calculations with multiple operators. One such calculation is the average increase or decrease in ridership during specific time intervals. By comparing ridership during peak and off-peak hours or weekdays versus weekends, it is possible to determine when the subway system is busiest and when it is underutilized.

Another useful calculation is the average percentage change in ridership between different stations or subway lines. This can help identify the most popular stations or lines and can inform decisions about where to allocate resources and infrastructure improvements.

Additionally, it is possible to explore the correlation between ridership and other factors such as weather, events, or holidays. By analyzing how ridership fluctuates in response to external factors, transportation planners can adjust schedules and routes to better serve the needs of riders.

Calculations with multiple operators can also reveal patterns in ridership based on time of year or day of the week. By analyzing the average change in ridership on different days or months, transportation planners can make informed decisions about scheduling and staffing.

Finally, regression analysis can be used to identify the overall trend in ridership over time. By examining long-term ridership trends, transportation planners can identify areas for improvement and plan for future infrastructure projects to better meet the needs of subway riders.

Learn more about Calculation

brainly.com/question/30151794

#SPJ11

Find the measures of angle a and B. Round to the nearest degree.

Answers

The measure of angle A and angle B are 28.61 and 61.39 rounded to two decimal place, using trigonometric ratio of tangent for the respective angles.

What is trigonometric ratios?

The trigonometric ratios is concerned with the relationship of an angle of a right-angled triangle to ratios of two side lengths.

The basic trigonometric ratios includes;

sine, cosine and tangent.

tan B = 6/11 {opposite/adjacent}

B= tan⁻¹(6/11) {cross multiplication}

B = 28.6105.

tan A = 11/6 {opposite/adjacent}

A= tan⁻¹(11/6) {cross multiplication}

A = 61.3895.

Therefore, the measure of angles A and B are 28.61 and 61.39 rounded to two decimal place, using trigonometric ratio of tangent for the respective angles.

Read more about trigonometric ratios here: https://brainly.com/question/11967894

#SPJ1

A survey of 7th and 8th grade student who were asked whether or not they were in favor of or against school uniforms. This two-way table shows the results. How many 7th grade students are against school uniforms?​

Answers

64 students were against school uniforms as shown in the table
Other Questions
in his first year, a math teacher earned $32,000. each successive year, he earned a 5% raise. what were his total earning over his 30-year career?\ An employee who was more worried about having a safe work environment and a financially sound pension plan than anything else would be most concerned with which of the needs in Maslow's hierarchy of needs? information for hobson corp. for the current year ($ in millions): income from continuing operations before tax$155 loss on discontinued operation (pretax) 32 temporary differences (all related to operating income): accrued warranty expense in excess of expense included in operating income 10 depreciation deducted on tax return in excess of depreciation expense 25 permanent differences (all related to operating income): nondeductible portion of entertainment expense 5 the applicable enacted tax rate for all periods is 25%. how much tax expense on income from continuing operations would be reported in hobson's income statement? What is the relative tax advantage of debt? Assume that personal and corporate taxes are given by TC= (corporate tax rate) = 35 percent; TpE = personal tax rate on equity income = 30 percent; and Tp = personal tax rate on interest income = 20 percent.a. 1.76b. 1.16c. 0.86d. 1.35 what is basin and range currently, tyra's physical activity level is sedentary. how would incorporating regular physical activity into her lifestyle affect tyra's diabetes management? multiple choice she will store more glycogen in her liver. she will not experience any changes because physical activity does not affect diabetes management. her insulin sensitivity will improve, which will assist with blood glucose regulation. she will experience hypoglycemia if she eats too many carbohydrates. Among Homer Simpson's many experiences as an (failed) entrepreneur is the time that he started a firm to recycle used grease. The current dividend on this stock is $0.90, and Homer expects that the dividend growth rate will be 6% for each of the next two years, 12% in the third year, and then will remain steady at 10% thereafter. Given the risky nature of purifying grease, prospective investors require a 15% rate of return if they are to invest in the stock of this enterprise. How much is this stock worth today? Based on the above information, how much should you expect the stock to cost in three years? Your company has $3,000,000 that can be used for triangular arbitrage. You observe the following exchange rates are the following:You can sell dollars for 0.888 euros per dollar and buy dollars for 0.896 euros per dollars.You can sell Australian dollars (A$) for $.73 and buy Australian dollars for $.75.You can sell Australian dollars (A$) for 0.68 euros per A$ and buy Australian dollars (A$) for 0.70 euros per A$.a. What profits can you earn from triangular arbitrage?b. Someone in the company is concerned about the plan to use triangular arbitrage like this, calling it a "risky scheme" that could backfire and hurt the profitability of the company. are they right? Explain why or why not. shaina and ian, who recently got married, bought a new house in florida and moved there from their previous homes in illinois. in the context of residence patterns, this is an example of the What is top-down design?- A top-down design is the decomposition of a system into smaller parts in order to comprehend its compositional sub-systems.-In top-down design, a system's overview is designed, specifying, yet not detailing any first-level subsystems.- A top-down design is also known as a stepwise design. The atmospheric pressure on top of Mt. Everest (elevation 29,028) is 250. torr. Calculate the atmospheric pressure in mmHg and atm. Round each of your answers to 3 significant digits. All else equal, properties with riskier expected future cash flowsa. sell at higher cap ratesb. have lower expected (internal) rates of returnc. sell at lower cap rates a patient has microcytic hypochromic anemia. which of the following pathogenic mechanisms may cause anemia in this patient? (select all that apply.) 1. increased basal metabolic rate 2. decreased erythrocyte life span 3. disturbances of the iron cycle 4. swelling in the tissues 5. failure of mechanisms of compensatory erythropoiesis If you horizontally WWII part three: On The Battlefield NEWSELA during its most recent period, superior manufacturing expected to incur the following costs: $420,000 of overhead, $700,000 of materials, and $280,000 in labor. superior applied overhead based on direct labor cost. actual production required an overhead cost of $413,000, $798,000 in materials used, and $308,000 in labor. all of the goods were completed. how much is the amount of over or underapplied overhead? select answer from the options below Many exoenzymes from pathogens are virulence factors. 1) True 2) False Modelo BETO: Qu piensas de esta camiseta? A mi me gusta mucho. BETO:pantalones son bonitos y el precio no es muy alto. KIKO: S, las dos cosas son geniales. BETO: Tambin me gustan_suteres que estn cerca de ti. De quetalla essuter negro, el de slo un color?KIKO: Mediano, y t quieres encontrar uno grande, no? Qu te parecenzapatos que estn en la pared de all? Ybotas?Necesito buscar o zapatos o botas porque los que tengo son muy viejos. BETO: Yo no necesito comprar ni zapatos ni botas pero me encantatraje que est al lado de los zapatos. Voy a preguntarle al dependientecunto cuestaB. Sra. Surez and her friend Lucy are in a shopping mall. Write the correctdemonstrative adjectives in the spaces provided, according to what each one is saying. SRA. SUAREZ: Qu te parecenbolsos de colores vivos queestn all?LUCY: A m me gusta msbolso de color pastel que esten aquella mesa. SRA. SUAREZ: Te gustara comprar-pantalones que tengo aqui?LUCY: No, son muy pequeos, Eh! Mira,camisa de telasinttica que lleva aquel seor! Qu bonita!SRA. SUAREZ: Oh, no! Entienda donde estamos no aceptantarjetas de crditoLUCY: No importa! Mira lo que tengo en la mano. cupnde regalo me lo dio mi hermano. how can firms best maintain competitive advantage? multiple choice question. by divesting themselves of the brand by copying competitors' brands, images, products, and promotions by reinforcing the brand image through merchandise, service, and promotion by using the cheapest supplier for all raw materials Porque El Principito nunca renunciaba a una pregunta